PT40.S3.Q23 - a television manufacturing plant

Barack Obama 2.0Barack Obama 2.0 Alum Member
edited August 2016 in Logical Reasoning 87 karma
I cannot for the life of me figure out why C and D are wrong, can give a coherent detailed explanation for why they're wrong?
https://7sage.com/lsat_explanations/lsat-40-section-3-question-23/

Comments

  • BinghamtonDaveBinghamtonDave Alum Member 🍌🍌
    edited August 2016 8700 karma
    Yes, This is a flaw question that is often times pretty frustrating. As we shall see, I think there is 2 flaws at play here.
    The first sentence is some really basic math. Out of 1,000 workers, we get an average of 10 absent each day. I don’t think it is merely a coincidence that this is 1%. The second sentence tells us that when exactly 10 are not at work, everything is “normal” as far as production goes at the plant: again we have this appeal to the 1%. What the author is trying to get across here is that when 990 workers are working that day, things are "normal.” From this information, the third sentence then states that the plant could cut 10 workers and this wouldn’t be a problem as far as production goes. The third sentence reeks with assurance from the author! Take a second to reread the third sentence: the author is trying to tell us that the plant won’t have any loss. Given the information provided, are we really to believe the author’s conclusion follows from the stated premises?

    Ok, what is the flaw(s) here? We will call them Flaw1 and Flaw2. Flaw1 to me appears to be that the author has failed to consider any nuisance with regard to who is absent on particular days vis-a-vis who they would be cutting. So if during a typical day: say, Thursday we had 10 absentees from the 10 disparate areas of the plant so as to really allow the areas of the plant to level-off with a -1 and continue on with productivity unabated, that’s fine. But that doesn’t mean that we can just cut 10 people as the author suggests and things will work great. What if all 10 of the cuts are in the box-packaging area of the television plant? And now boxing up all these televisions falls on the remaining people? The author seems to be ignoring that cuts would have to mirror absences in such a way as to give us the same results (the “normal rate.”) The author doesn’t pay attention to any of this.

    But wait, there is an even more fundamental flaw here: Flaw2 is that the author has yet to correct for the absentees! The author has failed to correct for that 1% that is either sick, or hung over or has jury duty! Are we to suppose that there is some pattern to who is absent? and no one else is ever absent? and we are cutting specifically these people? No… that would be totally unwarranted. Has the author even tried to tell us that absences will cease? no… By the author’s own logic, wouldn’t a single absence put us outside of 990?

    (A)No. Although literally true, this has no effect on our argument or the flaw(s).
    (B)Correct. This is Flaw2!
    (C)No. This actually states something that the argument never assumed. As JY points out in the explanation: the argument actually states “Normal—>990”. What the argument is assuming is something else. I think there is also something subtle going on with (C). If you get beyond the fact that the argument never assumed this, and you attribute this to the argument, it isn’t actually a description of the flaw. Both Flaw1 and Flaw2 are flaws that take place after the author has stated that the -10 absentees (990) are compatible with normal productivity. This answer choice just tells us that the author assumes this.
    (D)No. The argument does indeed do this. But this answer choice doesn’t tell us enough. This appears to be our Flaw1, but actually we are not pointing out much. We are not pointing out that those workers who the author overlooks are the ones being cut.
    (E)No. This actually isn’t something that the argument does. The whole point of the argument is a flawed attempt to say that as long as you stay with that -10 numbers: things will be the same.
  • Barack Obama 2.0Barack Obama 2.0 Alum Member
    edited August 2016 87 karma
    Your explanation for C was fantastic but just for clarification for answer choice D, is it wrong basically because we don't know which specific workers are being cut?
  • BinghamtonDaveBinghamtonDave Alum Member 🍌🍌
    edited August 2016 8700 karma
    @"Barack Obama 2.0" correct. (D) does a terrible job actually trying to recall Flaw1. All (D) tells us is that the author overlooks some workers whose jobs are "crucial." This is literally true. The author overlooks a lot of things. But in order to describe Flaw1 (D) would have to say something like: Overlooks the possibility that some really crucial workers might actually be cut in the -10 cuts. (D) simply acknowledges that these crucial workers exist. We need an answer choice that not only acknowledges that crucial workers exist, but says that these crucial ones might get caught in the -10 cuts.

    edit spelling 2:24 EST


  • dcdcdcdcdcdcdcdcdcdc Alum Member
    382 karma

    This one was rough. I pre-phrased the flaw exactly. Then B and E threw me and I couldn't recover. I went with E because I felt it encompassed the idea of an absentee rate reduction assumption. E references those "employed" and not those working. So E says the author assumes that the rate of production wouldn't be altered by changing employment. I think by stating that a reduced number of employees without a change in absentee rates does have some effect on production rates. However, the focus there is clearly on the absentee rate remaining unchanged and not on the total number of workers employed, which is why B is the correct choice. Moreover, E is a pretty broad statement. I think it is right to see the stimulus as stating that within this specific range we could maintain the rate; this says nothing like the broad relationship assumed in E that both factors are independent. The author doesn't need to assume E for their argument, just note that it isn't relevant for the specific scenario they are discussing.

    For me, this was really down to the tricky wording of the answer choices. :(

  • BinghamtonDaveBinghamtonDave Alum Member 🍌🍌
    8700 karma

    Thank you for brining this thread forward. This is an excellent question that I love revisiting.

Sign In or Register to comment.